LSAT and Law School Admissions Forum

Get expert LSAT preparation and law school admissions advice from PowerScore Test Preparation.

User avatar
 Jeff Wren
PowerScore Staff
  • PowerScore Staff
  • Posts: 578
  • Joined: Oct 19, 2022
|
#100743
Complete Question Explanation

Weaken. The correct answer choice is (C).

The stimulus contains the following argument:

Premise: Hospital staff must not deceive patients about their medical treatment according to the ethics code.

Premise: Dr. Faris gave medication A to a patient and informed him that it would help him sleep.

Premise: Medication A has no known sleep-inducing properties.

Counter-Premise: The patient's sleep did improve after taking medication A.

Conclusion: Dr. Faris violated the ethics code.

This argument involves causal reasoning, specifically, the difference between direct and indirect causes. Here, the ethicist seems to assume that, because medication A has no known sleep-inducing properties, then there would be no reason for Dr. Faris to believe that this medication would help his patient sleep. This is why the ethicist concludes that Dr. Faris has violated the code of ethics. For the ethicist, the fact that Dr. Faris said that medication A would help his patient sleep even though it doesn't directly induce sleep means that Dr. Faris has "deceived" his patient about the treatment.

The problem/flaw in this argument is that it may very well be possible for a medication to help a patient sleep without directly causing a patient to sleep. As anyone knows who has ever struggled falling asleep, there can be many things that prevent one from going to sleep (e.g. stress, anxiety, pain, discomfort, noise/light/distractions, being too hot/cold, etc.). Any medication that reduces one of these problems could indirectly help the patient fall asleep.

In this argument, the ethicist admits that the patient's sleep did improve after taking medication A, which undermines the ethicists argument. Of course, it's possible that the two things are just coincidental. It's also possible that medication A had a "placebo" effect, which might still qualify as a "deceptive" practice in the ethic's code.

In this question, we are asked to weaken the ethicist's argument. A possible prephrase would be that medication A had an indirect effect on improving the patient's sleep by reducing something that is preventing the patient from sleeping.

Answer choice (A): The fact that Dr. Faris was aware that medication A doesn't have sleep inducing properties doesn't weaken the argument. If anything, it would strengthen the argument by eliminating another possible way that Dr. Faris could have given the medication without "deceiving" the patient. Generally, the word "deceive" implies an intent to lie. If Dr. Faris were simply mistaken about the effects the medication A, that would probably not fall under "deception." This is an opposite answer.

Answer choice (B): The fact that a committee is currently considering revisions to the hospital's ethics code has no effect on the argument, which is limited to the issue of whether Dr. Faris violated the ethics code in its current form, not whether that code is correct or in need to revision. This is a shell wrong answer.

Answer choice (C): This is the correct answer choice. This answer provides a specific example of our prephrase. If the patient's pain/discomfort is preventing him from sleeping and this medication relieves that pain and lets the patient sleep more easily, then Dr. Faris was correct (and not being deceptive) in telling the patient that it would help him sleep.

Answer choice (D): The fact that other doctors prescribed medication A to patients who had trouble sleeping doesn't tell us whether Dr. Faris was being deceptive (and thereby violating the ethics code) without additional information. For example, it is possible that all of the doctors (including Dr. Faris) are being deceptive, and that there is an unspoken practice to give patients medication A to anyone complaining about sleep just to shut them up even though there is no reason to think that it works!

Answer choice (E): The fact that Dr. Faris knew that the patient wasn't taking any other medications that have sleep-inducing properties doesn't weaken the argument that Dr. Faris violated the ethics code by deceiving the patient. The only issue to resolve for this argument is whether Dr. Faris had reason to believe that medication A would help the patient sleep despite it not directly inducing sleep. This answer has no effect on this issue and the argument itself.
User avatar
 German.Steel
  • Posts: 55
  • Joined: Jun 12, 2021
|
#90969
I ultimately chose (C) but was rather uncomfortable trying to eliminate (D). It seems to me they are conceptually similar:

(C) provides a reason to think there was no "deception" involved - Dr. Faris wasn't deceiving, he knew that improved sleep would likely result from the med.

(D) also provides a reason to think there was no "deception" involved - with other doctors prescribing this medication to patients who had trouble sleeping, Dr. Faris was less likely to be "deceiving" and more likely to be simply going along with the typical prescriptions he/she has observed from other doctors treating patients who had trouble sleeping.

I recognize (D) is marginally more of a stretch, which is why I chose (C), but I'm rarely this uncomfortable on a LR question (let alone question 7) and would love any further insight on how to more confidently dispatch (D).

Thanks in advance!
 Robert Carroll
PowerScore Staff
  • PowerScore Staff
  • Posts: 1787
  • Joined: Dec 06, 2013
|
#91674
German.Steel,

There's a simple problem with answer choice (D). The other members of the staff would be doing the same thing as Dr. Faris, so they could just as easily be in violation of the ethics code. Other people did the same thing - why would that show the interpretation of Faris's action is incorrect rather than that the other people are also at fault? There is no way it makes it any less likely that Faris was being deceptive. "Other people also did this" immediately invites the response "and all of them were lying too."

Robert Carroll
User avatar
 mkarimi73
  • Posts: 73
  • Joined: Aug 18, 2022
|
#97742
Is (E) wrong for the same reasons as (D), Mr. Carroll, since the other medications are irrelevant to the conclusion at hand? Just checking.
User avatar
 atierney
PowerScore Staff
  • PowerScore Staff
  • Posts: 215
  • Joined: Jul 06, 2021
|
#98099
Yes, that is correct.

Get the most out of your LSAT Prep Plus subscription.

Analyze and track your performance with our Testing and Analytics Package.